Add missing figures and solutions to Serway & Jewett v8's chapter 29.
authorW. Trevor King <wking@tremily.us>
Thu, 31 May 2012 12:44:20 +0000 (08:44 -0400)
committerW. Trevor King <wking@tremily.us>
Thu, 31 May 2012 12:44:20 +0000 (08:44 -0400)
latex/problems/Serway_and_Jewett_8/problem29.02.tex
latex/problems/Serway_and_Jewett_8/problem29.03.tex
latex/problems/Serway_and_Jewett_8/problem29.22.tex
latex/problems/Serway_and_Jewett_8/problem29.31.tex
latex/problems/Serway_and_Jewett_8/problem29.37.tex
latex/problems/Serway_and_Jewett_8/problem29.40.tex
latex/problems/Serway_and_Jewett_8/problem29.57.tex
latex/problems/Serway_and_Jewett_8/problem29.61.tex

index 6c133b67068d105e2d6ef6a727f20f5036778b70..c54394d15c84cb72397bac2d606dfa318f985d47 100644 (file)
@@ -1,4 +1,4 @@
-\begin{problem*}{29.3}
+\begin{problem*}{29.2}
 Determine the initial direction of the deflection of charged particles
 as they enter the magnetic fields shown in Figure~P29.2.
 \begin{center}
@@ -32,7 +32,7 @@ Vector v = Velocity(a.center(), dir=0);
 v.draw();
 a.draw();
 
-label("\Part{a}", (0,0.5*height), N);
+label("\Part{a}", (0,-0.5*height), S);
 \end{asy}
 \hspace{\stretch{1}}
 \begin{asy}
@@ -60,7 +60,7 @@ Vector v = Velocity(a.center(), dir=180);
 v.draw();
 a.draw();
 
-label("\Part{b}", (0,0.5*height), N);
+label("\Part{b}", (0,-0.5*height), S);
 \end{asy}
 \hspace{\stretch{1}}
 \begin{asy}
@@ -89,7 +89,7 @@ Vector v = Velocity(a.center(), dir=180);
 v.draw();
 a.draw();
 
-label("\Part{c}", (0,0.5*height), N);
+label("\Part{c}", (0,-0.5*height), S);
 \end{asy}
 \hspace{\stretch{1}}
 \begin{asy}
@@ -126,11 +126,29 @@ Vector v = Velocity(a.center(), dir=90);
 v.draw();
 a.draw();
 
-label("\Part{d}", (0,0.5*height), N);
+label("\Part{d}", a.lc.center, S);
 \end{asy}
 \hspace{\stretch{1}}
+\rule{0pt}{0pt}
 \end{center}
 \end{problem*}
 
 \begin{solution}
+The force on a charged particle moving through a magnetic field is
+$\vect{F}=q\vect{v}\times\vect{B}$.  From Newton second law,
+$\vect{a}=\vect{F}/m=q/m\cdot\vect{v}\times\vect{B}$.  We can find the
+direction of deflection comes using this formula and the right hand
+rule.
+
+\Part{a}
+\ans{Up}.
+
+\Part{b}
+\ans{Out of the page}.
+
+\Part{c}
+\ans{No deflection}.
+
+\Part{d}
+\ans{Into the page}.
 \end{solution}
index b43eab8194213a1104a20a828260a257f363bf3d..fd89e833865435495e664ba1547c5eea56fe54fa 100644 (file)
@@ -41,6 +41,7 @@ v.draw();
 label("\Part{c}", (0,0), S);
 \end{asy}
 \hspace{\stretch{1}}
+\rule{0pt}{0pt}
 \end{center}
 \end{problem*}
 
index 1831a542519c16a3d0160f97f8fdc7e3529543f0..bc2162c6fb7014cd7b82ddb2f406d962630e66cf 100644 (file)
@@ -1,7 +1,7 @@
 \begin{problem*}{29.22}
 Assume the region to the right of a certain plain contains a uniform
 magnetic field of magnitude $1.00\U{mT}$ and the field is zero in the
-fregion to the left of the plane as shown in Figure~P29.22.  An
+region to the left of the plane as shown in Figure~P29.22.  An
 electron, originally traveling perpendicular to the boundary plane,
 passes into the region of the field.  \Part{a} Determine the time
 interval required for the electron to leave the ``field-filled''
@@ -19,21 +19,10 @@ real width = 2u;
 real dx = 0.49u;
 real dy = dx;
 
-Vector Bs[];
-int n = (int)(width / dx);
-int m = (int)(height / dy);
-real xstart = -width/2 + (dx+fmod(width,dx))/2.0;
-real ystart = -height/2 + (dy+fmod(height,dy))/2.0;
-for (int i=0; i<n; i+=1) {
-  for (int j=0; j<m; j+=1)  {
-    Bs.push(BField((xstart+i*dx, ystart+j*dy), phi=-90));
-  }
-}
-for (int i=0; i<Bs.length; i+=1) {
-  Bs[i].draw();
-}
+Vector v = BField(phi=-90);
+vector_field((0,0), width=width, height=height, v=v);
 
-Charge a = nCharge((-0.5*width-24pt, 0), "$e^-$");
+Charge a = nCharge((-0.5*width-24pt, 0), Label("$e^-$", align=S));
 Vector v = Velocity(a.center(), dir=0,
     L=Label("$\vect{v}$", position=EndPoint, align=RightSide));
 v.draw();
@@ -81,9 +70,11 @@ Now that they've given us $r$, we can find $v$ and $K$.
   r &= \frac{vm}{|q|B} \\
   v &= \frac{|q|Br}{m} \\
   K &= \frac{1}{2} m v^2 = \frac{1}{2} m \p({\frac{|q|Br}{m}})^2
-    = \frac{q^2 B^2 r^2}{2m}
-    = \ans{5.63\E{-18}\U{J}}
-    = 5.63\E{-18}\U{J} \cdot \frac{1\U{eV}}{1.6\E{-19}\U{J}}
-    = \ans{35.2\U{eV}} \;.
+    = \frac{(|q|Br)^2}{2m}
+    = \frac{(1.60\E{-19}\U{C}\cdot1.00\E{-3}\U{T}\cdot2.00\E{-2}\U{m})^2}
+           {2\cdot9.11\E{-31}\U{kg}}
+    = \ans{5.62\E{-18}\U{J}} \\
+    &= 5.62\E{-18}\U{J} \cdot \frac{1\U{eV}}{1.6\E{-19}\U{J}}
+    = \ans{35.1\U{eV}} \;.
 \end{align}
 \end{solution}
index c361e0a6861f016a883bb123388562085cce4a6d..e597bfc18bd54733021637b864c365cbfa29dd32 100644 (file)
@@ -8,4 +8,22 @@ which the current passes.
 \end{problem*}
 
 \begin{solution}
+\Part{a}
+Using the formula for magnetic force on a current carrying wire,
+\begin{align}
+  \vect{F} &= I\vect{L}\times\vect{B} \\
+  F &= ILB\sin(\theta) = ILB \;,
+\end{align}
+because $\theta=90\dg$.  Solving for $B$,
+\begin{equation}
+  B = \frac{F}{IL} = \frac{F/L}{I}
+    = \frac{0.120\U{N/m}}{15.0\U{A}}
+    = \ans{8.00\U{mT}} \;.
+\end{equation}
+
+\Part{b}
+Because $\vect{B}$ is perpendicular to $\vect{L}$, $\vect{L}$ points
+along the positive $x$ axis, and $\vect{F}=I\vect{L}\times\vect{B}$,
+$\vect{B}$ points in the $-y$ direction, $\vect{B}$ must point \ans{in
+  the positive $z$ direction}.
 \end{solution}
index 32d6af1d3adfacd5327b526930ac99a189bbb0f2..7d2062abeb28627346a768dd5ed3b984bdba0fd2 100644 (file)
@@ -6,7 +6,61 @@ the direction shown and rolls along the rails without slipping.  A
 uniform magnetic field of magnitude $0.240\U{T}$ is directed
 perpendicular to the rod and rails.  If it starts from rest, what is
 the speed of the rod as it leaves the rails?
+\begin{center}
+\begin{asy}
+import Mechanics;
+import ElectroMag;
+import Circ;
+
+real d = 2cm;
+real L = 3cm;
+real r = 6pt;
+
+Vector v = BField(phi=-90);
+vector_field(((L+r)/2,0), width=L-r, height=d, v=v);
+wire((0, d/2), (L, d/2));
+wire((0, -d/2), (L, -d/2));
+Block bar = Block(width=2*r, height=d, fill=yellow);  bar.draw();
+
+Distance Dr = Distance((0, -d/2), (r, -d/2), offset=3pt,
+    Label("$r$", align=S));
+Dr.draw();
+Distance Dd = Distance((0, -d/2), (0, d/2), offset=-24pt,
+    Label("$d=12.0\U{cm}$", align=W));
+Dd.draw();
+Distance DL = Distance((0, d/2), (L, d/2), offset=-12pt,
+    Label("$L=45.0\U{cm}$", align=N));
+DL.draw();
+
+draw((0, d/4)--(0, -d/4), CurrentPen, Arrow);
+label("$I$", (-r,0), align=W);
+\end{asy}
+\end{center}
 \end{problem*}
 
 \begin{solution}
+The magnetic force on the rod is
+\begin{align}
+  \vect{F}_B &= I\vect{d}\times{B} \\
+  F_B &= IdB\sin(\theta) = IdB \;.
+\end{align}
+After the rod rolls a distance $\vect{L}$ along the rails, this force
+has done
+\begin{equation}
+  W_B = \vect{F}_B \cdot \vect{L} = F_B L = IdBL
+\end{equation}
+of work on the rod.  Conserving energy,
+\begin{equation}
+  W_B = IdBL = K = \frac{1}{2} m v^2 + \frac{1}{2}I_m\omega^2 \;.
+\end{equation}
+To solve this, we need the moment of inertia for a cylinder about it's
+axis ($I_m=\frac{1}{2}mR^2$) and a relationship between $\omega$ and
+$v$ ($v=R\omega$).  The rest is algrebra:
+\begin{align}
+ IdBL &= \frac{1}{2} m v^2 + \frac{1}{4} m R^2 \cdot \p({\frac{v}{R}})^2
+   = \frac{3}{4} m v^2 \\
+ v^2 &= \frac{4IdBL}{3m} \\
+ v &= \sqrt{\frac{4IdBL}{3m}} = \ans{1.07\U{m/s}} \;.
+\end{align}
+This speed doesn't depend on the mass of the rod.
 \end{solution}
index 2f18fb506b387e436475b755c999837f8eccdb71..1e4417835d4e42c906137e67cfc22dfc3d22f523 100644 (file)
@@ -1,5 +1,5 @@
 \begin{problem*}{29.40}
-Consider the system pictured in Figure P29.40.  A $15.\U{cm}$
+Consider the system pictured in Figure P29.40.  A $15.0\U{cm}$
 horizontal wire of mass $15.0\U{g}$ is placed between two thin,
 vertical conductors, and a uniform magnetic field acts perpendicular
 to the page.  The wire is free to move vertically without friction on
@@ -11,7 +11,82 @@ move upward at a constant velocity.  \Part{c} Find the magnitude and
 direction of the minimum magnetic field required to move the wire at a
 constant speed.  \Part{d} What happens if the magnetic field exceeds
 this minimum value?
+\begin{center}
+% |     |  15.0 cm between verticals
+% +--<--+
+% |  5A |
+% |     |
+% v 5A  ^ 5A
+\begin{asy}
+import Mechanics;
+import Circ;
+
+real u = 1cm;
+
+pair ul = (-u, u);    // upper left
+pair ml = (-u, 0);    // middle left
+pair ll = (-u, -2u);  // lower left
+pair ur = (u, u);     // upper right
+pair mr = (u, 0);
+pair lr = (u, -2u);
+
+MultiTerminal I = current(label=Label("$5.00\U{A}$"), draw=false);
+I.centerto(lr, mr);  I.label.align = E;  I.draw();
+wire(lr, I.terminal[0]);
+wire(I.terminal[1], ur);
+I.centerto(mr, ml);  I.label.align = S;  I.draw();
+wire(mr, I.terminal[0]);
+wire(I.terminal[1], ml);
+I.centerto(ml, ll);  I.label.align = W;  I.draw();
+wire(ul, I.terminal[0]);
+wire(I.terminal[1], ll);
+dot(mr);
+dot(ml);
+
+Distance D = Distance(ml, mr, offset=-u/2, Label("$15.0\U{cm}$", align=N));
+D.draw();
+\end{asy}
+\end{center}
 \end{problem*}
 
 \begin{solution}
+\Part{a}
+The only forces acting on the horizontal wire are \ans{gravity and the
+force of the magnetic field on the current carried by the wire}.
+
+\Part{b}
+If the wire is moving at a constant velocity,
+\begin{align}
+  0 &= \deriv{t}{\vect{v}} = \vect{a} = \frac{\vect{F}}{m} \\
+  \ans{\vect{F}} &= \ans{0} \;.
+\end{align}
+In English, this means the net force on the wire must be zero.
+Because there are only two forces acting on the wire, this means these
+forces must be equal in magnitude and opposite in direction.
+
+\Part{c}
+The gravitational force on the wire will be $F_g=mg$ directed
+downwards, and the magnetic force on the wire will be
+$F_B=I\vect{L}\times\vect{B}$ directed upwards.  Because the magnetic
+field is perpendicular to the page, and the current is directed along
+the page, the angle between $\vect{L}$ and $\vect{B}$ is $90\dg$, so
+$F_B=ILB\sin{90\dg}=ILB$.  Summing forces in the vertical direction,
+\begin{align}
+  0 &= \sum_i F_{i,y} = F_B - F_g = ILB - mg \\
+  ILB &= mg \\
+  B &= \frac{mg}{IL}
+    = \frac{0.0150\U{kg} \cdot 9.80\U{m/s$^2$}}{5.00\U{A} \cdot 0.150\U{m}}
+    = \ans{196\U{mT}} \;.
+\end{align}
+In order to generate an upwards force on a leftwards current, the
+magnetic field must be directed \ans{out of the page}.
+
+\Part{d}
+If the magnetic field exceeds this value, \ans{the rod will accelerate
+  upwards}, moving up faster and faster.  If the magnetic field is
+less than this value, the rod will accelerate downwards, moving up
+slower and slower, eventually pausing, and then moving back down
+faster and faster.  At the critical magnetic field, where the forces
+balance, the rod will keep moving forever at it's initial velocity (or
+hang in place if the initial velocity was zero).
 \end{solution}
index fa6f9f49686ef9f8ff3387ff272223707653a18b..e0821aa029b9a892cd8b7281a3e9d2db50c93aca 100644 (file)
@@ -1,6 +1,91 @@
 \begin{problem*}{29.57}
-The upper poretion of 
+The upper portion of the circuit in Figure~P29.57 is fixed.  The
+horizontal wire at the bottom has a mass of $10.0\U{g}$ and is
+$5.00\U{cm}$ long.  This wire hangs in the gravitational field of the
+Earth from identical light springs connected to the upper portion of
+the circuit.  The springs stretch $0.500\U{cm}$ under the weight of
+the wire, and the circuit has a total resistance of $12.0\U{\Ohm}$.
+When a magnetic field is turned on, directed out of the page, the
+springs stretch an additional $0.300\U{cm}$.  Only the horizontal wire
+at the bottom of the circuit is in the magnetic field.  What is the
+magnitude of the magnetic field?
+\begin{center}
+%          24.0V
+% +--/\/\/--|i--+
+% |             |
+% Sp            Sp
+% |<-- 5.00 cm->|
+% +-------------+
+%        Bout
+%
+\begin{asy}
+import Mechanics;
+import ElectroMag;
+import Circ;
+
+real u = 1cm;
+
+MultiTerminal Bat = battery(dir=180, label=Label("$24.0\U{V}$", align=N));
+MultiTerminal R = resistor(Bat.terminal[1], dir=180);
+Spring Sl = Spring(R.terminal[1] - (0,u), R.terminal[1] - (0,2u),
+    deadLength=1mm, unstretchedLength=u, L=Label("$k$", embed=Shift));
+Sl.draw();
+Spring Sr = Spring(Bat.terminal[0] - (0,u), Bat.terminal[0] - (0,2u),
+    deadLength=1mm, unstretchedLength=u, L=Label("$k$", embed=Shift));
+Sr.draw();
+
+Vector v = BField(phi=90);
+real w = Sr.pTo.x - Sl.pTo.x;
+vector_field((Sl.pTo + Sr.pTo)/2 + (0, -u), width=1.5*w, height=u, v=v);
+
+Distance DL = Distance(Sl.pTo, Sr.pTo, offset=u/3,
+    Label("$5.00\U{cm}$", align=N));
+DL.draw();
+
+wire(R.terminal[1], Sl.pFrom);
+wire(Bat.terminal[0], Sr.pFrom);
+wire(Sl.pTo, Sr.pTo, udsq, dist=-u);
+\end{asy}
+\end{center}
 \end{problem*}
 
 \begin{solution}
+We have two situations here, one where $B=0$ and $\Delta
+x=0.500\U{cm}$ (call this $B_1$ and $\Delta x_1$), and another where
+$B$ is unknown and $\Delta x=\Delta x_1 + 0.300\U{cm}=0.800\U{cm}$
+(call this $B_2$ and $\Delta x_2$).  Because both are in equilibrium,
+the net force on the hanging wire must be zero in each case.  The only
+forces acting on the wire are gravity pulling down with $F_g=mg$, two
+springs pulling up with $F_s=k\Delta x$ (for each spring), and the magnetic
+field pulling down with $F_B=ILB\cos(90\dg)$.  Summing forces in the
+vertical direction for the first situation,
+\begin{align}
+  0 &= \sum_i F_{i,y} = 2k\Delta x_1 - mg \\
+  2k\Delta x_1 &= mg \\
+  k &= \frac{mg}{2\Delta x_1} \;.
+\end{align}
+We can plug this into the sum of vertical forces for the second
+situation,
+\begin{align}
+  0 &= \sum_i F_{i,y} = 2k\Delta x_2 - mg - ILB_2 \\
+  ILB_2 &= 2k\Delta x_2 - mg
+    = 2\frac{mg}{2\Delta x_1}\Delta x_2 - mg
+    = mg\p({\frac{\Delta x_2}{\Delta x_1} - 1}) \\
+  B_2 &= \frac{mg}{IL}\p({\frac{\Delta x_2}{\Delta x_1} - 1}) \;.
+\end{align}
+This looks good, but we don't have a value for the current $I$.  Using
+a Kirchhoff loop starting behind the battery and going counter
+clockwise,
+\begin{align}
+  0 &= V - IR \\
+  I &= \frac{V}{R} \;.
+\end{align}
+Plugging in for $B_2$,
+\begin{equation}
+  B_2 = \frac{mgR}{VL}\p({\frac{\Delta x_2}{\Delta x_1} - 1})
+    = \frac{0.0100\U{kg} \cdot 9.80\U{m/s$^2$} \cdot 12.0\U{\Ohm}}
+           {24.0\U{V} \cdot 0.0500\U{m}}
+      \p({\frac{0.800\U{cm}}{0.500\U{cm}} - 1})
+    = \ans{588\U{mT}} \;.
+\end{equation}
 \end{solution}
index 6c1cf45b993b3b7dfcc88ab3fee009e881444633..97a1358f9f0be935133a320a93411e51ecec7361 100644 (file)
@@ -6,4 +6,41 @@ field is required to keep the rod moving at a constant speed?
 \end{problem*}
 
 \begin{solution}
+This is just like Problem~29.40, except the force balancing the
+magnetic force is friction instead of gravity.  Drawing a free-body
+diagram looking from the side,
+\begin{center}
+\begin{asy}
+import Mechanics;
+
+real mag = 1cm;
+real mu = 0.3;
+
+Vector F_B = Force(mag=mu*mag, Label("$F_B$", position=EndPoint));
+Vector F_g = Force(mag=mag, dir=-90, Label("$F_g$", position=EndPoint));
+Vector F_N = Force(mag=mag, dir=90, Label("$F_N$", position=EndPoint));
+Vector F_f = Force(mag=mu*mag, dir=180, Label("$F_f$", position=EndPoint));
+F_B.draw();
+F_g.draw();
+F_N.draw();
+F_f.draw();
+dot((0,0));
+\end{asy}
+\end{center}
+
+Summing the forces in the vertical direction,
+\begin{align}
+  0 &= \sum_i F_{i,y} = F_N - F_g = F_N - mg \\
+  F_N &= mg \\
+\end{align}
+Now that we know the normal force, we can find the force of friction
+and sum the forces in the horizontal direction.
+\begin{align}
+  0 &= \sum_i F_{i,x} = F_B - F_f = ILB - \mu F_N = ILB\sin(90\dg) - \mu mg \\
+  ILB &= \mu mg \\
+  B &= \frac{\mu mg}{IL}
+    = \frac{0.100 \cdot 0.200\U{kg} \cdot 9.80\U{m/s$^2$}}
+           {10.0\U{A} \cdot 0.500\U{m}}
+    = \ans{39.2\U{mT}} \;.
+\end{align}
 \end{solution}